How can I find a cubic function with specific local maximum and minimum values?

  • Thread starter Thread starter ryan.1015
  • Start date Start date
  • Tags Tags
    Cubic Function
Click For Summary
To find a cubic function g(x) = ax^3 + bx^2 + cx + d with a local maximum of 3 at x = -7 and a local minimum of -9 at x = 12, the derivative g'(x) must equal zero at these points. The initial derivative can be expressed as g'(x) = A(x + 7)(x - 12), where A is a constant. The values g(-7) = 3 and g(12) = -9 must be used to solve for the coefficients a, b, c, and d. Additionally, the second derivative g''(x) can provide insights into the nature of the critical points. Careful attention to notation and function names is essential for clarity in the calculations.
ryan.1015
Messages
14
Reaction score
0

Homework Statement


find a cubic function g(x)=ax^3 +bx^2+cx +d that has a local maximum value of 3 at -7 and a local minimum value 0f -9 at 12.


Homework Equations





The Attempt at a Solution


I know the derivative should equal zero for a max or min to occure. So i got f '(x)=(x+7)(x-12). then i got F '(x)=x^2-5x-84 and plugged that into the original equation. I got A=1/3 b=5/2 and c=-84. I'm not sure how to fet D, or if i did this first part right
 
Physics news on Phys.org
Basically you know that g(3)=-7 and g(-9)=12. And also that g'(3)=g'(9)=0. Solve now.


EDIT: It should be g(7)=-3 and g(12)=9, not the other way around
 
Last edited:
You switch function names, with the original starting out as g(x).

g'(x) doesn't have to be exactly (x + 7)(x - 12). It could be a constant multiple of this expression, namely g'(x) = A(x + 7)(x - 12) = A(x^2 - 5x - 84). Also, you can calculate g'(x) from the original equation for g, and compare this to the one above.

You know that g(-7) = 3 and that g(12) = -9.

What about the second derivative? You can calculate g''(x) from the equation above, as well as from the original equation. What do you know about the value of the second derivative at a local maximum? At a local minimum?

You should be more careful with your notation. You have referred to the original function as g, f, and F. Also, the coefficients of the equation for g(x) involved a, b, c, and d, not A.
 
Question: A clock's minute hand has length 4 and its hour hand has length 3. What is the distance between the tips at the moment when it is increasing most rapidly?(Putnam Exam Question) Answer: Making assumption that both the hands moves at constant angular velocities, the answer is ## \sqrt{7} .## But don't you think this assumption is somewhat doubtful and wrong?

Similar threads

  • · Replies 2 ·
Replies
2
Views
2K
  • · Replies 5 ·
Replies
5
Views
2K
Replies
30
Views
3K
  • · Replies 58 ·
2
Replies
58
Views
4K
Replies
11
Views
2K
Replies
5
Views
2K
  • · Replies 4 ·
Replies
4
Views
2K
Replies
5
Views
3K
  • · Replies 18 ·
Replies
18
Views
3K
  • · Replies 18 ·
Replies
18
Views
5K